Đến nội dung

Hình ảnh

$\sum_{i=1}^kx_i^2<\frac{1}{2}\sum _{i=1}^kx_i<\frac{1}{4}\sum _{i=1}^kx_i^3$

- - - - -

  • Please log in to reply
Chủ đề này có 1 trả lời

#1
the unknown

the unknown

    Thượng sĩ

  • Thành viên
  • 208 Bài viết

Giả sử ta có các số thực dương $x_1,x_2,...,x_k$ thỏa mãn điều kiện:

$x_1^2+x_2^2+...+x_k^2<\frac{x_1+x_2+...+x_k}{2}$

$x_1+x_2+...+x_k<\frac{x_1^3+x_2^3+...+x_k^3}{2}$

Tìm giá trị nhỏ nhất của $k$ và với giá trị $k$ đó, hãy chỉ ra một bộ số $(x_1,x_2,...,x_k)$ thỏa mãn điều kiện trên.


Bài viết đã được chỉnh sửa nội dung bởi the unknown: 03-08-2016 - 19:19

$\texttt{If you don't know where you are going, any road will get you there}$


#2
nhungvienkimcuong

nhungvienkimcuong

    Thiếu úy

  • Hiệp sỹ
  • 669 Bài viết

Bài này gớm thật, đáp số là $516$ (xem ở đây)


Đừng khóc vì chuyện đã kết thúc hãy cười vì chuyện đã xảy ra ~O) 
Thật kì lạ anh không thể nhớ đến tên mình mà chỉ nhớ đến tên em :wub:
Chúa tạo ra vũ trụ của con người còn em tạo ra vũ trụ của anh :ukliam2:





2 người đang xem chủ đề

0 thành viên, 2 khách, 0 thành viên ẩn danh